Proving a certain holomorphic function is polinomic












0












$begingroup$


Suppose we have an holomorphic function $f$ on the open unit disk $D(0,1)$ s.t.:



$$forall rin (0,1) exists nin mathbb{N}| max_{C(0,r)}|f|=r^n$$



prove that $f$ is polinomic. Honestly I don't know where to start, every approach I take leads me to triviality or useless formulas.










share|cite|improve this question









$endgroup$

















    0












    $begingroup$


    Suppose we have an holomorphic function $f$ on the open unit disk $D(0,1)$ s.t.:



    $$forall rin (0,1) exists nin mathbb{N}| max_{C(0,r)}|f|=r^n$$



    prove that $f$ is polinomic. Honestly I don't know where to start, every approach I take leads me to triviality or useless formulas.










    share|cite|improve this question









    $endgroup$















      0












      0








      0





      $begingroup$


      Suppose we have an holomorphic function $f$ on the open unit disk $D(0,1)$ s.t.:



      $$forall rin (0,1) exists nin mathbb{N}| max_{C(0,r)}|f|=r^n$$



      prove that $f$ is polinomic. Honestly I don't know where to start, every approach I take leads me to triviality or useless formulas.










      share|cite|improve this question









      $endgroup$




      Suppose we have an holomorphic function $f$ on the open unit disk $D(0,1)$ s.t.:



      $$forall rin (0,1) exists nin mathbb{N}| max_{C(0,r)}|f|=r^n$$



      prove that $f$ is polinomic. Honestly I don't know where to start, every approach I take leads me to triviality or useless formulas.







      complex-analysis holomorphic-functions cauchy-integral-formula maximum-principle






      share|cite|improve this question













      share|cite|improve this question











      share|cite|improve this question




      share|cite|improve this question










      asked Dec 10 '18 at 10:29









      Renato FaraoneRenato Faraone

      2,33111627




      2,33111627






















          1 Answer
          1






          active

          oldest

          votes


















          2












          $begingroup$

          First note that $f$ has a zero of $n$-th order at $z=0$. Let $g(z)=frac{f(z)}{z^n}$ and observe that $g$ attains maximum modulus in the interior.



          $textbf{EDIT:}$ I've misread the problem. We cannot draw directly from the statement the fact that there is an independent $n$ such that $max_{|z|=r} |f(z)|=r^n$ holds. What we have is $n(r)geq 1$ such that $$ max_{|z|=r}|f(z)|=r^{n(r)}.$$To see there is independent $n$, note that
          $$
          phi(r) = max_{|z|=r}|f(z)|
          $$
          is continuous in $rin(0,1)$ by the continuity of $f$. Thus, a positive integer-valued function
          $$
          n(r) = frac{phi(r)}{log r}
          $$
          is also continuous, establishing $n(r) equiv n$. If you feel this argument tricky, you can solve the problem inductively, by noting that if there are $r_1<r_2$ such that $n(r_1)=n(r_2)=m$ attains minimum of $n(cdot)$, then we can argue by maximum modulus principle that $frac{f(z)}{z^m}$ is a constant.






          share|cite|improve this answer











          $endgroup$













          • $begingroup$
            I can't see why it has a zero at the origin
            $endgroup$
            – Renato Faraone
            Dec 10 '18 at 11:18






          • 1




            $begingroup$
            Just take $rto 0$ and use continuity. Divide $f$ by $z$ and do the same thing $n$ times.
            $endgroup$
            – Song
            Dec 10 '18 at 11:21










          • $begingroup$
            Why does $g$ attains its maximum modulus in the interior?
            $endgroup$
            – Renato Faraone
            Dec 10 '18 at 11:33






          • 1




            $begingroup$
            It is because $max_{|z|=r} |g(z)|=1$ for $0<r<1$. I'll explain it in more detail. So there is $w$ with $|w|=frac{1}{3}$ such that $|g(w)|=1=max_{|z|=frac{1}{2}}|g(z)|$. Now, apply Maximum modulus principle.
            $endgroup$
            – Song
            Dec 10 '18 at 11:39










          • $begingroup$
            @Song The hypothesis says for each $r$ there exist $n$ such that $cdots$; so $n$ varies with $r$. Aren't you assuming that $n$ is independent of $r$?.
            $endgroup$
            – Kavi Rama Murthy
            Dec 11 '18 at 7:46













          Your Answer





          StackExchange.ifUsing("editor", function () {
          return StackExchange.using("mathjaxEditing", function () {
          StackExchange.MarkdownEditor.creationCallbacks.add(function (editor, postfix) {
          StackExchange.mathjaxEditing.prepareWmdForMathJax(editor, postfix, [["$", "$"], ["\\(","\\)"]]);
          });
          });
          }, "mathjax-editing");

          StackExchange.ready(function() {
          var channelOptions = {
          tags: "".split(" "),
          id: "69"
          };
          initTagRenderer("".split(" "), "".split(" "), channelOptions);

          StackExchange.using("externalEditor", function() {
          // Have to fire editor after snippets, if snippets enabled
          if (StackExchange.settings.snippets.snippetsEnabled) {
          StackExchange.using("snippets", function() {
          createEditor();
          });
          }
          else {
          createEditor();
          }
          });

          function createEditor() {
          StackExchange.prepareEditor({
          heartbeatType: 'answer',
          autoActivateHeartbeat: false,
          convertImagesToLinks: true,
          noModals: true,
          showLowRepImageUploadWarning: true,
          reputationToPostImages: 10,
          bindNavPrevention: true,
          postfix: "",
          imageUploader: {
          brandingHtml: "Powered by u003ca class="icon-imgur-white" href="https://imgur.com/"u003eu003c/au003e",
          contentPolicyHtml: "User contributions licensed under u003ca href="https://creativecommons.org/licenses/by-sa/3.0/"u003ecc by-sa 3.0 with attribution requiredu003c/au003e u003ca href="https://stackoverflow.com/legal/content-policy"u003e(content policy)u003c/au003e",
          allowUrls: true
          },
          noCode: true, onDemand: true,
          discardSelector: ".discard-answer"
          ,immediatelyShowMarkdownHelp:true
          });


          }
          });














          draft saved

          draft discarded


















          StackExchange.ready(
          function () {
          StackExchange.openid.initPostLogin('.new-post-login', 'https%3a%2f%2fmath.stackexchange.com%2fquestions%2f3033748%2fproving-a-certain-holomorphic-function-is-polinomic%23new-answer', 'question_page');
          }
          );

          Post as a guest















          Required, but never shown

























          1 Answer
          1






          active

          oldest

          votes








          1 Answer
          1






          active

          oldest

          votes









          active

          oldest

          votes






          active

          oldest

          votes









          2












          $begingroup$

          First note that $f$ has a zero of $n$-th order at $z=0$. Let $g(z)=frac{f(z)}{z^n}$ and observe that $g$ attains maximum modulus in the interior.



          $textbf{EDIT:}$ I've misread the problem. We cannot draw directly from the statement the fact that there is an independent $n$ such that $max_{|z|=r} |f(z)|=r^n$ holds. What we have is $n(r)geq 1$ such that $$ max_{|z|=r}|f(z)|=r^{n(r)}.$$To see there is independent $n$, note that
          $$
          phi(r) = max_{|z|=r}|f(z)|
          $$
          is continuous in $rin(0,1)$ by the continuity of $f$. Thus, a positive integer-valued function
          $$
          n(r) = frac{phi(r)}{log r}
          $$
          is also continuous, establishing $n(r) equiv n$. If you feel this argument tricky, you can solve the problem inductively, by noting that if there are $r_1<r_2$ such that $n(r_1)=n(r_2)=m$ attains minimum of $n(cdot)$, then we can argue by maximum modulus principle that $frac{f(z)}{z^m}$ is a constant.






          share|cite|improve this answer











          $endgroup$













          • $begingroup$
            I can't see why it has a zero at the origin
            $endgroup$
            – Renato Faraone
            Dec 10 '18 at 11:18






          • 1




            $begingroup$
            Just take $rto 0$ and use continuity. Divide $f$ by $z$ and do the same thing $n$ times.
            $endgroup$
            – Song
            Dec 10 '18 at 11:21










          • $begingroup$
            Why does $g$ attains its maximum modulus in the interior?
            $endgroup$
            – Renato Faraone
            Dec 10 '18 at 11:33






          • 1




            $begingroup$
            It is because $max_{|z|=r} |g(z)|=1$ for $0<r<1$. I'll explain it in more detail. So there is $w$ with $|w|=frac{1}{3}$ such that $|g(w)|=1=max_{|z|=frac{1}{2}}|g(z)|$. Now, apply Maximum modulus principle.
            $endgroup$
            – Song
            Dec 10 '18 at 11:39










          • $begingroup$
            @Song The hypothesis says for each $r$ there exist $n$ such that $cdots$; so $n$ varies with $r$. Aren't you assuming that $n$ is independent of $r$?.
            $endgroup$
            – Kavi Rama Murthy
            Dec 11 '18 at 7:46


















          2












          $begingroup$

          First note that $f$ has a zero of $n$-th order at $z=0$. Let $g(z)=frac{f(z)}{z^n}$ and observe that $g$ attains maximum modulus in the interior.



          $textbf{EDIT:}$ I've misread the problem. We cannot draw directly from the statement the fact that there is an independent $n$ such that $max_{|z|=r} |f(z)|=r^n$ holds. What we have is $n(r)geq 1$ such that $$ max_{|z|=r}|f(z)|=r^{n(r)}.$$To see there is independent $n$, note that
          $$
          phi(r) = max_{|z|=r}|f(z)|
          $$
          is continuous in $rin(0,1)$ by the continuity of $f$. Thus, a positive integer-valued function
          $$
          n(r) = frac{phi(r)}{log r}
          $$
          is also continuous, establishing $n(r) equiv n$. If you feel this argument tricky, you can solve the problem inductively, by noting that if there are $r_1<r_2$ such that $n(r_1)=n(r_2)=m$ attains minimum of $n(cdot)$, then we can argue by maximum modulus principle that $frac{f(z)}{z^m}$ is a constant.






          share|cite|improve this answer











          $endgroup$













          • $begingroup$
            I can't see why it has a zero at the origin
            $endgroup$
            – Renato Faraone
            Dec 10 '18 at 11:18






          • 1




            $begingroup$
            Just take $rto 0$ and use continuity. Divide $f$ by $z$ and do the same thing $n$ times.
            $endgroup$
            – Song
            Dec 10 '18 at 11:21










          • $begingroup$
            Why does $g$ attains its maximum modulus in the interior?
            $endgroup$
            – Renato Faraone
            Dec 10 '18 at 11:33






          • 1




            $begingroup$
            It is because $max_{|z|=r} |g(z)|=1$ for $0<r<1$. I'll explain it in more detail. So there is $w$ with $|w|=frac{1}{3}$ such that $|g(w)|=1=max_{|z|=frac{1}{2}}|g(z)|$. Now, apply Maximum modulus principle.
            $endgroup$
            – Song
            Dec 10 '18 at 11:39










          • $begingroup$
            @Song The hypothesis says for each $r$ there exist $n$ such that $cdots$; so $n$ varies with $r$. Aren't you assuming that $n$ is independent of $r$?.
            $endgroup$
            – Kavi Rama Murthy
            Dec 11 '18 at 7:46
















          2












          2








          2





          $begingroup$

          First note that $f$ has a zero of $n$-th order at $z=0$. Let $g(z)=frac{f(z)}{z^n}$ and observe that $g$ attains maximum modulus in the interior.



          $textbf{EDIT:}$ I've misread the problem. We cannot draw directly from the statement the fact that there is an independent $n$ such that $max_{|z|=r} |f(z)|=r^n$ holds. What we have is $n(r)geq 1$ such that $$ max_{|z|=r}|f(z)|=r^{n(r)}.$$To see there is independent $n$, note that
          $$
          phi(r) = max_{|z|=r}|f(z)|
          $$
          is continuous in $rin(0,1)$ by the continuity of $f$. Thus, a positive integer-valued function
          $$
          n(r) = frac{phi(r)}{log r}
          $$
          is also continuous, establishing $n(r) equiv n$. If you feel this argument tricky, you can solve the problem inductively, by noting that if there are $r_1<r_2$ such that $n(r_1)=n(r_2)=m$ attains minimum of $n(cdot)$, then we can argue by maximum modulus principle that $frac{f(z)}{z^m}$ is a constant.






          share|cite|improve this answer











          $endgroup$



          First note that $f$ has a zero of $n$-th order at $z=0$. Let $g(z)=frac{f(z)}{z^n}$ and observe that $g$ attains maximum modulus in the interior.



          $textbf{EDIT:}$ I've misread the problem. We cannot draw directly from the statement the fact that there is an independent $n$ such that $max_{|z|=r} |f(z)|=r^n$ holds. What we have is $n(r)geq 1$ such that $$ max_{|z|=r}|f(z)|=r^{n(r)}.$$To see there is independent $n$, note that
          $$
          phi(r) = max_{|z|=r}|f(z)|
          $$
          is continuous in $rin(0,1)$ by the continuity of $f$. Thus, a positive integer-valued function
          $$
          n(r) = frac{phi(r)}{log r}
          $$
          is also continuous, establishing $n(r) equiv n$. If you feel this argument tricky, you can solve the problem inductively, by noting that if there are $r_1<r_2$ such that $n(r_1)=n(r_2)=m$ attains minimum of $n(cdot)$, then we can argue by maximum modulus principle that $frac{f(z)}{z^m}$ is a constant.







          share|cite|improve this answer














          share|cite|improve this answer



          share|cite|improve this answer








          edited Dec 11 '18 at 9:03

























          answered Dec 10 '18 at 11:05









          SongSong

          11.3k628




          11.3k628












          • $begingroup$
            I can't see why it has a zero at the origin
            $endgroup$
            – Renato Faraone
            Dec 10 '18 at 11:18






          • 1




            $begingroup$
            Just take $rto 0$ and use continuity. Divide $f$ by $z$ and do the same thing $n$ times.
            $endgroup$
            – Song
            Dec 10 '18 at 11:21










          • $begingroup$
            Why does $g$ attains its maximum modulus in the interior?
            $endgroup$
            – Renato Faraone
            Dec 10 '18 at 11:33






          • 1




            $begingroup$
            It is because $max_{|z|=r} |g(z)|=1$ for $0<r<1$. I'll explain it in more detail. So there is $w$ with $|w|=frac{1}{3}$ such that $|g(w)|=1=max_{|z|=frac{1}{2}}|g(z)|$. Now, apply Maximum modulus principle.
            $endgroup$
            – Song
            Dec 10 '18 at 11:39










          • $begingroup$
            @Song The hypothesis says for each $r$ there exist $n$ such that $cdots$; so $n$ varies with $r$. Aren't you assuming that $n$ is independent of $r$?.
            $endgroup$
            – Kavi Rama Murthy
            Dec 11 '18 at 7:46




















          • $begingroup$
            I can't see why it has a zero at the origin
            $endgroup$
            – Renato Faraone
            Dec 10 '18 at 11:18






          • 1




            $begingroup$
            Just take $rto 0$ and use continuity. Divide $f$ by $z$ and do the same thing $n$ times.
            $endgroup$
            – Song
            Dec 10 '18 at 11:21










          • $begingroup$
            Why does $g$ attains its maximum modulus in the interior?
            $endgroup$
            – Renato Faraone
            Dec 10 '18 at 11:33






          • 1




            $begingroup$
            It is because $max_{|z|=r} |g(z)|=1$ for $0<r<1$. I'll explain it in more detail. So there is $w$ with $|w|=frac{1}{3}$ such that $|g(w)|=1=max_{|z|=frac{1}{2}}|g(z)|$. Now, apply Maximum modulus principle.
            $endgroup$
            – Song
            Dec 10 '18 at 11:39










          • $begingroup$
            @Song The hypothesis says for each $r$ there exist $n$ such that $cdots$; so $n$ varies with $r$. Aren't you assuming that $n$ is independent of $r$?.
            $endgroup$
            – Kavi Rama Murthy
            Dec 11 '18 at 7:46


















          $begingroup$
          I can't see why it has a zero at the origin
          $endgroup$
          – Renato Faraone
          Dec 10 '18 at 11:18




          $begingroup$
          I can't see why it has a zero at the origin
          $endgroup$
          – Renato Faraone
          Dec 10 '18 at 11:18




          1




          1




          $begingroup$
          Just take $rto 0$ and use continuity. Divide $f$ by $z$ and do the same thing $n$ times.
          $endgroup$
          – Song
          Dec 10 '18 at 11:21




          $begingroup$
          Just take $rto 0$ and use continuity. Divide $f$ by $z$ and do the same thing $n$ times.
          $endgroup$
          – Song
          Dec 10 '18 at 11:21












          $begingroup$
          Why does $g$ attains its maximum modulus in the interior?
          $endgroup$
          – Renato Faraone
          Dec 10 '18 at 11:33




          $begingroup$
          Why does $g$ attains its maximum modulus in the interior?
          $endgroup$
          – Renato Faraone
          Dec 10 '18 at 11:33




          1




          1




          $begingroup$
          It is because $max_{|z|=r} |g(z)|=1$ for $0<r<1$. I'll explain it in more detail. So there is $w$ with $|w|=frac{1}{3}$ such that $|g(w)|=1=max_{|z|=frac{1}{2}}|g(z)|$. Now, apply Maximum modulus principle.
          $endgroup$
          – Song
          Dec 10 '18 at 11:39




          $begingroup$
          It is because $max_{|z|=r} |g(z)|=1$ for $0<r<1$. I'll explain it in more detail. So there is $w$ with $|w|=frac{1}{3}$ such that $|g(w)|=1=max_{|z|=frac{1}{2}}|g(z)|$. Now, apply Maximum modulus principle.
          $endgroup$
          – Song
          Dec 10 '18 at 11:39












          $begingroup$
          @Song The hypothesis says for each $r$ there exist $n$ such that $cdots$; so $n$ varies with $r$. Aren't you assuming that $n$ is independent of $r$?.
          $endgroup$
          – Kavi Rama Murthy
          Dec 11 '18 at 7:46






          $begingroup$
          @Song The hypothesis says for each $r$ there exist $n$ such that $cdots$; so $n$ varies with $r$. Aren't you assuming that $n$ is independent of $r$?.
          $endgroup$
          – Kavi Rama Murthy
          Dec 11 '18 at 7:46




















          draft saved

          draft discarded




















































          Thanks for contributing an answer to Mathematics Stack Exchange!


          • Please be sure to answer the question. Provide details and share your research!

          But avoid



          • Asking for help, clarification, or responding to other answers.

          • Making statements based on opinion; back them up with references or personal experience.


          Use MathJax to format equations. MathJax reference.


          To learn more, see our tips on writing great answers.




          draft saved


          draft discarded














          StackExchange.ready(
          function () {
          StackExchange.openid.initPostLogin('.new-post-login', 'https%3a%2f%2fmath.stackexchange.com%2fquestions%2f3033748%2fproving-a-certain-holomorphic-function-is-polinomic%23new-answer', 'question_page');
          }
          );

          Post as a guest















          Required, but never shown





















































          Required, but never shown














          Required, but never shown












          Required, but never shown







          Required, but never shown

































          Required, but never shown














          Required, but never shown












          Required, but never shown







          Required, but never shown







          Popular posts from this blog

          Quarter-circle Tiles

          build a pushdown automaton that recognizes the reverse language of a given pushdown automaton?

          Mont Emei